Why isn't it B?
In Str w Nec Assumption questions, are we trying to validate the argument as it was given or make...
nardine6@my.yorku.ca on April 1, 2023
  • June 2002 LSAT
  • SEC4
  • Q17
2
Replies
Correct Answer
Can you explain how D would account for the six percent decrease? Upon seeing D I assumed this wo...
Sallyanne-Tejan on July 27, 2022
  • June 2002 LSAT
  • SEC4
  • Q5
1
Reply
How is D the answer?
That clearly supports his argument.
Tyler808 on July 25, 2022
  • June 2002 LSAT
  • SEC4
  • Q13
2
Replies
Why B is not correct?
I was struggling between B and E. I thought both of them are match with the structure of the argu...
Karry-Jiao on February 7, 2022
  • June 2002 LSAT
  • SEC4
  • Q14
5
Replies
C vs D
Why D and not C?
Kyland on January 3, 2022
  • June 2002 LSAT
  • SEC4
  • Q11
1
Reply
Answer choice B
I am lost on how B is the answer
amarachicynthia on May 24, 2021
  • June 2002 LSAT
  • SEC4
  • Q21
1
Reply
How is A different from E
If the desire for warmth and comfort is "nearly as strong as" their desire for food, doesn't that...
ilovethelsat on February 9, 2021
  • June 2002 LSAT
  • SEC4
  • Q6
2
Replies
June2000-S4-Q21
Please explain the correct answer? Thank you
maonuo on September 10, 2020
  • June 2002 LSAT
  • SEC4
  • Q21
1
Reply
Why is C incorrect?
Why is C incorrect?
Shiyi-Zhang on August 7, 2020
  • June 2002 LSAT
  • SEC4
  • Q2
2
Replies
Can you help me with this?
Why is it C
kzman29 on July 8, 2020
  • June 2002 LSAT
  • SEC4
  • Q9
1
Reply
Explanation
I was totally confused with this one. Please explain the passage and the answer. Thanks.
avif on July 6, 2020
  • June 2002 LSAT
  • SEC4
  • Q19
3
Replies
Could you please show me where I made the error...
I arrived at the opposite of B: decrease if that nation DOES NOT come to have a high level of eco...
yckim2180 on June 19, 2020
  • June 2002 LSAT
  • SEC4
  • Q8
1
Reply
answer?
B
Lucas on January 26, 2020
  • June 2002 LSAT
  • SEC4
  • Q8
1
Reply
Choice E
What about E makes it incorrect?
Meredith on January 9, 2020
  • June 2002 LSAT
  • SEC4
  • Q1
1
Reply
Please Explain
How can we deduce that C is the correct answer? Where can the reader inference that if a crim...
AnnaC on January 5, 2020
  • June 2002 LSAT
  • SEC4
  • Q1
2
Replies
Why not A?
I was debating between A and B but originally chose A. Please explain why A is wrong. Thank you
farnoushsalimian on October 29, 2019
  • June 2002 LSAT
  • SEC4
  • Q25
1
Reply
Help
Can you please explain the method of reasoning in the passage and why answer choice "E" is correct?
Maurice on May 10, 2019
  • June 2002 LSAT
  • SEC4
  • Q14
3
Replies
Why is D incorrect?
Why is D incorrect?
Shiyi-Zhang on May 7, 2019
  • June 2002 LSAT
  • SEC4
  • Q12
1
Reply
Please explain
The passage talks about old stones in Ireland and since a particular stone was discovered in Scot...
dallman0303 on October 22, 2015
  • June 2002 LSAT
  • SEC4
  • Q22
1
Reply
E appears to be invalid?
The passage does not discuss the number of fatalities. It only speaks to a significant reduction...
dallman0303 on October 15, 2015
  • June 2002 LSAT
  • SEC4
  • Q17
1
Reply